Solve by means of Laplace transforms

Click For Summary
SUMMARY

The discussion focuses on solving the differential equation y'' + 6y' + 13y = 0 using Laplace transforms. Participants emphasize the importance of applying the Laplace transform to both sides of the equation, specifically addressing the transforms of y'' and y'. The solution process involves expressing these transforms in terms of L(y), rearranging the equation, and then finding the inverse Laplace transform while incorporating the initial conditions y(0) = 3 and y'(0) = 7.

PREREQUISITES
  • Understanding of Laplace transforms and their properties
  • Familiarity with differential equations
  • Knowledge of initial value problems
  • Ability to perform inverse Laplace transforms
NEXT STEPS
  • Study the properties of Laplace transforms in detail
  • Practice solving linear differential equations using Laplace transforms
  • Learn about initial value problems and their solutions
  • Explore inverse Laplace transform techniques and applications
USEFUL FOR

Students and professionals in mathematics, engineering, and physics who are looking to deepen their understanding of solving differential equations using Laplace transforms.

chrisking2021
Messages
7
Reaction score
0
1. Please see attachment for Question I've put it into microsoft eqn editor.


2. I have not got a clue where to begin i don't want anyone to do it for me but some information on how i can do it for myself would be appreciated.


3. as above
 

Attachments

Last edited:
Physics news on Phys.org
Here is the question for those who don't want to risk infection or are lazy to download and open:
y''+6y'+13y = 0, y(0) = 3, y'(0) = 7.


Well, you start by applying the Laplace transform to both the LHS and RHS. What is Laplace transform of y'', y' ? Express it in terms of L(y) and then throw all the other terms to the RHS, and find the inverse Laplace transform of RHS. Then plug in the initial value conditions.
 
Question: A clock's minute hand has length 4 and its hour hand has length 3. What is the distance between the tips at the moment when it is increasing most rapidly?(Putnam Exam Question) Answer: Making assumption that both the hands moves at constant angular velocities, the answer is ## \sqrt{7} .## But don't you think this assumption is somewhat doubtful and wrong?

Similar threads

  • · Replies 1 ·
Replies
1
Views
1K
  • · Replies 7 ·
Replies
7
Views
2K
  • · Replies 8 ·
Replies
8
Views
3K
Replies
9
Views
2K
  • · Replies 2 ·
Replies
2
Views
2K
  • · Replies 4 ·
Replies
4
Views
3K
  • · Replies 14 ·
Replies
14
Views
2K
  • · Replies 3 ·
Replies
3
Views
2K
  • · Replies 9 ·
Replies
9
Views
3K
  • · Replies 8 ·
Replies
8
Views
2K